Proof of an integral property












3












$begingroup$


$$int_0^1 f(x)g'(x)dx=pi$$ If $f(1)g(1)=f(0)g(0)$ then $$int_0^1 f'(x)g(x)dx= -pi$$
So I have to prove this and I have absolutely no idea how to do it. I am guessing I will have to use the fundamental theorem of calculus and it show that the rate of change is $1$ because it didn't change from $f(1)g(1)$ to $f(0)g(0)$










share|cite|improve this question









New contributor




adam hany is a new contributor to this site. Take care in asking for clarification, commenting, and answering.
Check out our Code of Conduct.







$endgroup$








  • 5




    $begingroup$
    Do you know integration by parts?
    $endgroup$
    – Minus One-Twelfth
    2 hours ago


















3












$begingroup$


$$int_0^1 f(x)g'(x)dx=pi$$ If $f(1)g(1)=f(0)g(0)$ then $$int_0^1 f'(x)g(x)dx= -pi$$
So I have to prove this and I have absolutely no idea how to do it. I am guessing I will have to use the fundamental theorem of calculus and it show that the rate of change is $1$ because it didn't change from $f(1)g(1)$ to $f(0)g(0)$










share|cite|improve this question









New contributor




adam hany is a new contributor to this site. Take care in asking for clarification, commenting, and answering.
Check out our Code of Conduct.







$endgroup$








  • 5




    $begingroup$
    Do you know integration by parts?
    $endgroup$
    – Minus One-Twelfth
    2 hours ago
















3












3








3





$begingroup$


$$int_0^1 f(x)g'(x)dx=pi$$ If $f(1)g(1)=f(0)g(0)$ then $$int_0^1 f'(x)g(x)dx= -pi$$
So I have to prove this and I have absolutely no idea how to do it. I am guessing I will have to use the fundamental theorem of calculus and it show that the rate of change is $1$ because it didn't change from $f(1)g(1)$ to $f(0)g(0)$










share|cite|improve this question









New contributor




adam hany is a new contributor to this site. Take care in asking for clarification, commenting, and answering.
Check out our Code of Conduct.







$endgroup$




$$int_0^1 f(x)g'(x)dx=pi$$ If $f(1)g(1)=f(0)g(0)$ then $$int_0^1 f'(x)g(x)dx= -pi$$
So I have to prove this and I have absolutely no idea how to do it. I am guessing I will have to use the fundamental theorem of calculus and it show that the rate of change is $1$ because it didn't change from $f(1)g(1)$ to $f(0)g(0)$







calculus integration definite-integrals






share|cite|improve this question









New contributor




adam hany is a new contributor to this site. Take care in asking for clarification, commenting, and answering.
Check out our Code of Conduct.











share|cite|improve this question









New contributor




adam hany is a new contributor to this site. Take care in asking for clarification, commenting, and answering.
Check out our Code of Conduct.









share|cite|improve this question




share|cite|improve this question








edited 2 hours ago









Eevee Trainer

6,38811237




6,38811237






New contributor




adam hany is a new contributor to this site. Take care in asking for clarification, commenting, and answering.
Check out our Code of Conduct.









asked 2 hours ago









adam hanyadam hany

161




161




New contributor




adam hany is a new contributor to this site. Take care in asking for clarification, commenting, and answering.
Check out our Code of Conduct.





New contributor





adam hany is a new contributor to this site. Take care in asking for clarification, commenting, and answering.
Check out our Code of Conduct.






adam hany is a new contributor to this site. Take care in asking for clarification, commenting, and answering.
Check out our Code of Conduct.








  • 5




    $begingroup$
    Do you know integration by parts?
    $endgroup$
    – Minus One-Twelfth
    2 hours ago
















  • 5




    $begingroup$
    Do you know integration by parts?
    $endgroup$
    – Minus One-Twelfth
    2 hours ago










5




5




$begingroup$
Do you know integration by parts?
$endgroup$
– Minus One-Twelfth
2 hours ago






$begingroup$
Do you know integration by parts?
$endgroup$
– Minus One-Twelfth
2 hours ago












2 Answers
2






active

oldest

votes


















4












$begingroup$

Hint:



Utilize integration by parts:



$$int f(x)g'(x)dx = f(x)g(x) - int f'(x)g(x)$$



If we have a definite integral, then this formula becomes



$$int_a^b f(x)g'(x)dx = f(b)g(b) - f(a)g(a) - int_a^b f'(x)g(x)$$






share|cite|improve this answer









$endgroup$









  • 1




    $begingroup$
    thank you so much for the answer ∫baf(x)g′(x)dx=−∫baf′(x)g(x)
    $endgroup$
    – adam hany
    2 hours ago



















1












$begingroup$

Hint



$dfrac{d(f(x)cdot g(x))}{dx}=?$



Integrate both sides wrt $x$ between $[0,1]$






share|cite|improve this answer









$endgroup$













    Your Answer





    StackExchange.ifUsing("editor", function () {
    return StackExchange.using("mathjaxEditing", function () {
    StackExchange.MarkdownEditor.creationCallbacks.add(function (editor, postfix) {
    StackExchange.mathjaxEditing.prepareWmdForMathJax(editor, postfix, [["$", "$"], ["\\(","\\)"]]);
    });
    });
    }, "mathjax-editing");

    StackExchange.ready(function() {
    var channelOptions = {
    tags: "".split(" "),
    id: "69"
    };
    initTagRenderer("".split(" "), "".split(" "), channelOptions);

    StackExchange.using("externalEditor", function() {
    // Have to fire editor after snippets, if snippets enabled
    if (StackExchange.settings.snippets.snippetsEnabled) {
    StackExchange.using("snippets", function() {
    createEditor();
    });
    }
    else {
    createEditor();
    }
    });

    function createEditor() {
    StackExchange.prepareEditor({
    heartbeatType: 'answer',
    autoActivateHeartbeat: false,
    convertImagesToLinks: true,
    noModals: true,
    showLowRepImageUploadWarning: true,
    reputationToPostImages: 10,
    bindNavPrevention: true,
    postfix: "",
    imageUploader: {
    brandingHtml: "Powered by u003ca class="icon-imgur-white" href="https://imgur.com/"u003eu003c/au003e",
    contentPolicyHtml: "User contributions licensed under u003ca href="https://creativecommons.org/licenses/by-sa/3.0/"u003ecc by-sa 3.0 with attribution requiredu003c/au003e u003ca href="https://stackoverflow.com/legal/content-policy"u003e(content policy)u003c/au003e",
    allowUrls: true
    },
    noCode: true, onDemand: true,
    discardSelector: ".discard-answer"
    ,immediatelyShowMarkdownHelp:true
    });


    }
    });






    adam hany is a new contributor. Be nice, and check out our Code of Conduct.










    draft saved

    draft discarded


















    StackExchange.ready(
    function () {
    StackExchange.openid.initPostLogin('.new-post-login', 'https%3a%2f%2fmath.stackexchange.com%2fquestions%2f3120855%2fproof-of-an-integral-property%23new-answer', 'question_page');
    }
    );

    Post as a guest















    Required, but never shown

























    2 Answers
    2






    active

    oldest

    votes








    2 Answers
    2






    active

    oldest

    votes









    active

    oldest

    votes






    active

    oldest

    votes









    4












    $begingroup$

    Hint:



    Utilize integration by parts:



    $$int f(x)g'(x)dx = f(x)g(x) - int f'(x)g(x)$$



    If we have a definite integral, then this formula becomes



    $$int_a^b f(x)g'(x)dx = f(b)g(b) - f(a)g(a) - int_a^b f'(x)g(x)$$






    share|cite|improve this answer









    $endgroup$









    • 1




      $begingroup$
      thank you so much for the answer ∫baf(x)g′(x)dx=−∫baf′(x)g(x)
      $endgroup$
      – adam hany
      2 hours ago
















    4












    $begingroup$

    Hint:



    Utilize integration by parts:



    $$int f(x)g'(x)dx = f(x)g(x) - int f'(x)g(x)$$



    If we have a definite integral, then this formula becomes



    $$int_a^b f(x)g'(x)dx = f(b)g(b) - f(a)g(a) - int_a^b f'(x)g(x)$$






    share|cite|improve this answer









    $endgroup$









    • 1




      $begingroup$
      thank you so much for the answer ∫baf(x)g′(x)dx=−∫baf′(x)g(x)
      $endgroup$
      – adam hany
      2 hours ago














    4












    4








    4





    $begingroup$

    Hint:



    Utilize integration by parts:



    $$int f(x)g'(x)dx = f(x)g(x) - int f'(x)g(x)$$



    If we have a definite integral, then this formula becomes



    $$int_a^b f(x)g'(x)dx = f(b)g(b) - f(a)g(a) - int_a^b f'(x)g(x)$$






    share|cite|improve this answer









    $endgroup$



    Hint:



    Utilize integration by parts:



    $$int f(x)g'(x)dx = f(x)g(x) - int f'(x)g(x)$$



    If we have a definite integral, then this formula becomes



    $$int_a^b f(x)g'(x)dx = f(b)g(b) - f(a)g(a) - int_a^b f'(x)g(x)$$







    share|cite|improve this answer












    share|cite|improve this answer



    share|cite|improve this answer










    answered 2 hours ago









    Eevee TrainerEevee Trainer

    6,38811237




    6,38811237








    • 1




      $begingroup$
      thank you so much for the answer ∫baf(x)g′(x)dx=−∫baf′(x)g(x)
      $endgroup$
      – adam hany
      2 hours ago














    • 1




      $begingroup$
      thank you so much for the answer ∫baf(x)g′(x)dx=−∫baf′(x)g(x)
      $endgroup$
      – adam hany
      2 hours ago








    1




    1




    $begingroup$
    thank you so much for the answer ∫baf(x)g′(x)dx=−∫baf′(x)g(x)
    $endgroup$
    – adam hany
    2 hours ago




    $begingroup$
    thank you so much for the answer ∫baf(x)g′(x)dx=−∫baf′(x)g(x)
    $endgroup$
    – adam hany
    2 hours ago











    1












    $begingroup$

    Hint



    $dfrac{d(f(x)cdot g(x))}{dx}=?$



    Integrate both sides wrt $x$ between $[0,1]$






    share|cite|improve this answer









    $endgroup$


















      1












      $begingroup$

      Hint



      $dfrac{d(f(x)cdot g(x))}{dx}=?$



      Integrate both sides wrt $x$ between $[0,1]$






      share|cite|improve this answer









      $endgroup$
















        1












        1








        1





        $begingroup$

        Hint



        $dfrac{d(f(x)cdot g(x))}{dx}=?$



        Integrate both sides wrt $x$ between $[0,1]$






        share|cite|improve this answer









        $endgroup$



        Hint



        $dfrac{d(f(x)cdot g(x))}{dx}=?$



        Integrate both sides wrt $x$ between $[0,1]$







        share|cite|improve this answer












        share|cite|improve this answer



        share|cite|improve this answer










        answered 2 hours ago









        lab bhattacharjeelab bhattacharjee

        226k15157275




        226k15157275






















            adam hany is a new contributor. Be nice, and check out our Code of Conduct.










            draft saved

            draft discarded


















            adam hany is a new contributor. Be nice, and check out our Code of Conduct.













            adam hany is a new contributor. Be nice, and check out our Code of Conduct.












            adam hany is a new contributor. Be nice, and check out our Code of Conduct.
















            Thanks for contributing an answer to Mathematics Stack Exchange!


            • Please be sure to answer the question. Provide details and share your research!

            But avoid



            • Asking for help, clarification, or responding to other answers.

            • Making statements based on opinion; back them up with references or personal experience.


            Use MathJax to format equations. MathJax reference.


            To learn more, see our tips on writing great answers.




            draft saved


            draft discarded














            StackExchange.ready(
            function () {
            StackExchange.openid.initPostLogin('.new-post-login', 'https%3a%2f%2fmath.stackexchange.com%2fquestions%2f3120855%2fproof-of-an-integral-property%23new-answer', 'question_page');
            }
            );

            Post as a guest















            Required, but never shown





















































            Required, but never shown














            Required, but never shown












            Required, but never shown







            Required, but never shown

































            Required, but never shown














            Required, but never shown












            Required, but never shown







            Required, but never shown







            Popular posts from this blog

            サソリ

            広島県道265号伴広島線

            Accessing regular linux commands in Huawei's Dopra Linux